Checking for discontinuity of $f(x)= sqrt{(1+p^2)/(1+q^2)}$ if $x = p/q neq 0$, $f(x) = x$ otherwise.











up vote
-2
down vote

favorite












Let $f$ maps from $mathbb{R}$ to $mathbb{R}$. Be given by



$$f(x)=
begin{cases}
sqrt{(1+p^2)/(1+q^2)},& x=p/q neq 0,\
x, & text{otherwise}.
end{cases}
$$

Then



a) $f$ is continuous on $big((0,infty) cap mathbb{Q'}big) cup {0,1}$. (Note that $mathbb{Q}'$ denote $mathbb{Q}$ complement).



b) If $a in (-infty,0)$ then $lim_{xto a} f(x)$ does not exists.



c) If $a in [0,infty)$, then $lim_{xto a} f(x)=a$



I know it is discontinuous, and in the answer sheet a, b, c are correct options. But I confused about option (b), how limit doesn't exist for negative number and also confused about option (a), how it is continuous on 0. I have tried a lot, eventually leads to headache, please help me, I shall be very thankful for it. Thanks in advance.










share|cite|improve this question
























  • I have edited the formatting, let me know if the question is still accurate. Also, when you define $f(x)$ for $x = p/q$, are you assuming that $p/q$ is a rational with no common factors between $p$ and $q$?
    – Brahadeesh
    Nov 20 at 5:32










  • No such thing is mentioned, this question belongs to a test series of csir net
    – user532616
    Nov 20 at 15:47










  • Hmm, that’s odd...
    – Brahadeesh
    Nov 20 at 16:08










  • Option (b) is true for this reason: suppose $a < 0$ and $x to a$. If $x$ is rational, then $f(x)$ is positive and if $x$ is irrational then $f(x)$ is negative. So, the limit cannot exist (you need to complete the argument, but it should be easy from here).
    – Brahadeesh
    Nov 21 at 6:01















up vote
-2
down vote

favorite












Let $f$ maps from $mathbb{R}$ to $mathbb{R}$. Be given by



$$f(x)=
begin{cases}
sqrt{(1+p^2)/(1+q^2)},& x=p/q neq 0,\
x, & text{otherwise}.
end{cases}
$$

Then



a) $f$ is continuous on $big((0,infty) cap mathbb{Q'}big) cup {0,1}$. (Note that $mathbb{Q}'$ denote $mathbb{Q}$ complement).



b) If $a in (-infty,0)$ then $lim_{xto a} f(x)$ does not exists.



c) If $a in [0,infty)$, then $lim_{xto a} f(x)=a$



I know it is discontinuous, and in the answer sheet a, b, c are correct options. But I confused about option (b), how limit doesn't exist for negative number and also confused about option (a), how it is continuous on 0. I have tried a lot, eventually leads to headache, please help me, I shall be very thankful for it. Thanks in advance.










share|cite|improve this question
























  • I have edited the formatting, let me know if the question is still accurate. Also, when you define $f(x)$ for $x = p/q$, are you assuming that $p/q$ is a rational with no common factors between $p$ and $q$?
    – Brahadeesh
    Nov 20 at 5:32










  • No such thing is mentioned, this question belongs to a test series of csir net
    – user532616
    Nov 20 at 15:47










  • Hmm, that’s odd...
    – Brahadeesh
    Nov 20 at 16:08










  • Option (b) is true for this reason: suppose $a < 0$ and $x to a$. If $x$ is rational, then $f(x)$ is positive and if $x$ is irrational then $f(x)$ is negative. So, the limit cannot exist (you need to complete the argument, but it should be easy from here).
    – Brahadeesh
    Nov 21 at 6:01













up vote
-2
down vote

favorite









up vote
-2
down vote

favorite











Let $f$ maps from $mathbb{R}$ to $mathbb{R}$. Be given by



$$f(x)=
begin{cases}
sqrt{(1+p^2)/(1+q^2)},& x=p/q neq 0,\
x, & text{otherwise}.
end{cases}
$$

Then



a) $f$ is continuous on $big((0,infty) cap mathbb{Q'}big) cup {0,1}$. (Note that $mathbb{Q}'$ denote $mathbb{Q}$ complement).



b) If $a in (-infty,0)$ then $lim_{xto a} f(x)$ does not exists.



c) If $a in [0,infty)$, then $lim_{xto a} f(x)=a$



I know it is discontinuous, and in the answer sheet a, b, c are correct options. But I confused about option (b), how limit doesn't exist for negative number and also confused about option (a), how it is continuous on 0. I have tried a lot, eventually leads to headache, please help me, I shall be very thankful for it. Thanks in advance.










share|cite|improve this question















Let $f$ maps from $mathbb{R}$ to $mathbb{R}$. Be given by



$$f(x)=
begin{cases}
sqrt{(1+p^2)/(1+q^2)},& x=p/q neq 0,\
x, & text{otherwise}.
end{cases}
$$

Then



a) $f$ is continuous on $big((0,infty) cap mathbb{Q'}big) cup {0,1}$. (Note that $mathbb{Q}'$ denote $mathbb{Q}$ complement).



b) If $a in (-infty,0)$ then $lim_{xto a} f(x)$ does not exists.



c) If $a in [0,infty)$, then $lim_{xto a} f(x)=a$



I know it is discontinuous, and in the answer sheet a, b, c are correct options. But I confused about option (b), how limit doesn't exist for negative number and also confused about option (a), how it is continuous on 0. I have tried a lot, eventually leads to headache, please help me, I shall be very thankful for it. Thanks in advance.







calculus continuity






share|cite|improve this question















share|cite|improve this question













share|cite|improve this question




share|cite|improve this question








edited Nov 20 at 5:30









Brahadeesh

5,83441958




5,83441958










asked Nov 18 at 18:20









user532616

214




214












  • I have edited the formatting, let me know if the question is still accurate. Also, when you define $f(x)$ for $x = p/q$, are you assuming that $p/q$ is a rational with no common factors between $p$ and $q$?
    – Brahadeesh
    Nov 20 at 5:32










  • No such thing is mentioned, this question belongs to a test series of csir net
    – user532616
    Nov 20 at 15:47










  • Hmm, that’s odd...
    – Brahadeesh
    Nov 20 at 16:08










  • Option (b) is true for this reason: suppose $a < 0$ and $x to a$. If $x$ is rational, then $f(x)$ is positive and if $x$ is irrational then $f(x)$ is negative. So, the limit cannot exist (you need to complete the argument, but it should be easy from here).
    – Brahadeesh
    Nov 21 at 6:01


















  • I have edited the formatting, let me know if the question is still accurate. Also, when you define $f(x)$ for $x = p/q$, are you assuming that $p/q$ is a rational with no common factors between $p$ and $q$?
    – Brahadeesh
    Nov 20 at 5:32










  • No such thing is mentioned, this question belongs to a test series of csir net
    – user532616
    Nov 20 at 15:47










  • Hmm, that’s odd...
    – Brahadeesh
    Nov 20 at 16:08










  • Option (b) is true for this reason: suppose $a < 0$ and $x to a$. If $x$ is rational, then $f(x)$ is positive and if $x$ is irrational then $f(x)$ is negative. So, the limit cannot exist (you need to complete the argument, but it should be easy from here).
    – Brahadeesh
    Nov 21 at 6:01
















I have edited the formatting, let me know if the question is still accurate. Also, when you define $f(x)$ for $x = p/q$, are you assuming that $p/q$ is a rational with no common factors between $p$ and $q$?
– Brahadeesh
Nov 20 at 5:32




I have edited the formatting, let me know if the question is still accurate. Also, when you define $f(x)$ for $x = p/q$, are you assuming that $p/q$ is a rational with no common factors between $p$ and $q$?
– Brahadeesh
Nov 20 at 5:32












No such thing is mentioned, this question belongs to a test series of csir net
– user532616
Nov 20 at 15:47




No such thing is mentioned, this question belongs to a test series of csir net
– user532616
Nov 20 at 15:47












Hmm, that’s odd...
– Brahadeesh
Nov 20 at 16:08




Hmm, that’s odd...
– Brahadeesh
Nov 20 at 16:08












Option (b) is true for this reason: suppose $a < 0$ and $x to a$. If $x$ is rational, then $f(x)$ is positive and if $x$ is irrational then $f(x)$ is negative. So, the limit cannot exist (you need to complete the argument, but it should be easy from here).
– Brahadeesh
Nov 21 at 6:01




Option (b) is true for this reason: suppose $a < 0$ and $x to a$. If $x$ is rational, then $f(x)$ is positive and if $x$ is irrational then $f(x)$ is negative. So, the limit cannot exist (you need to complete the argument, but it should be easy from here).
– Brahadeesh
Nov 21 at 6:01















active

oldest

votes











Your Answer





StackExchange.ifUsing("editor", function () {
return StackExchange.using("mathjaxEditing", function () {
StackExchange.MarkdownEditor.creationCallbacks.add(function (editor, postfix) {
StackExchange.mathjaxEditing.prepareWmdForMathJax(editor, postfix, [["$", "$"], ["\\(","\\)"]]);
});
});
}, "mathjax-editing");

StackExchange.ready(function() {
var channelOptions = {
tags: "".split(" "),
id: "69"
};
initTagRenderer("".split(" "), "".split(" "), channelOptions);

StackExchange.using("externalEditor", function() {
// Have to fire editor after snippets, if snippets enabled
if (StackExchange.settings.snippets.snippetsEnabled) {
StackExchange.using("snippets", function() {
createEditor();
});
}
else {
createEditor();
}
});

function createEditor() {
StackExchange.prepareEditor({
heartbeatType: 'answer',
convertImagesToLinks: true,
noModals: true,
showLowRepImageUploadWarning: true,
reputationToPostImages: 10,
bindNavPrevention: true,
postfix: "",
imageUploader: {
brandingHtml: "Powered by u003ca class="icon-imgur-white" href="https://imgur.com/"u003eu003c/au003e",
contentPolicyHtml: "User contributions licensed under u003ca href="https://creativecommons.org/licenses/by-sa/3.0/"u003ecc by-sa 3.0 with attribution requiredu003c/au003e u003ca href="https://stackoverflow.com/legal/content-policy"u003e(content policy)u003c/au003e",
allowUrls: true
},
noCode: true, onDemand: true,
discardSelector: ".discard-answer"
,immediatelyShowMarkdownHelp:true
});


}
});














draft saved

draft discarded


















StackExchange.ready(
function () {
StackExchange.openid.initPostLogin('.new-post-login', 'https%3a%2f%2fmath.stackexchange.com%2fquestions%2f3003909%2fchecking-for-discontinuity-of-fx-sqrt1p2-1q2-if-x-p-q-neq-0%23new-answer', 'question_page');
}
);

Post as a guest















Required, but never shown






























active

oldest

votes













active

oldest

votes









active

oldest

votes






active

oldest

votes
















draft saved

draft discarded




















































Thanks for contributing an answer to Mathematics Stack Exchange!


  • Please be sure to answer the question. Provide details and share your research!

But avoid



  • Asking for help, clarification, or responding to other answers.

  • Making statements based on opinion; back them up with references or personal experience.


Use MathJax to format equations. MathJax reference.


To learn more, see our tips on writing great answers.





Some of your past answers have not been well-received, and you're in danger of being blocked from answering.


Please pay close attention to the following guidance:


  • Please be sure to answer the question. Provide details and share your research!

But avoid



  • Asking for help, clarification, or responding to other answers.

  • Making statements based on opinion; back them up with references or personal experience.


To learn more, see our tips on writing great answers.




draft saved


draft discarded














StackExchange.ready(
function () {
StackExchange.openid.initPostLogin('.new-post-login', 'https%3a%2f%2fmath.stackexchange.com%2fquestions%2f3003909%2fchecking-for-discontinuity-of-fx-sqrt1p2-1q2-if-x-p-q-neq-0%23new-answer', 'question_page');
}
);

Post as a guest















Required, but never shown





















































Required, but never shown














Required, but never shown












Required, but never shown







Required, but never shown

































Required, but never shown














Required, but never shown












Required, but never shown







Required, but never shown







Popular posts from this blog

Quarter-circle Tiles

build a pushdown automaton that recognizes the reverse language of a given pushdown automaton?

Mont Emei